Choosing weights of assignments to ensure that some particular students fail, while some other pass.












1












$begingroup$


Please have a look at my solution to the following problem.



enter image description here



Firstly, I think that there are two ways to interpret this problem. If we need to make sure all students in $mathscr{L}$ pass and all students in $mathscr{D}$ fail, this problem doesn't need to be a linear program since the constraints are already enough to determine the solutions (i.e. the average grades of the favorite students are at least 60%, and those of the disliked students less than 60%).



But there are of course situations where the above constraints could not all be met (e.g. a favorite student scoring lower than a disliked student in all 12 assignments), so I think the objective of the program should be to choose the weights so that as many students in $mathscr{L}$ pass as possible, and as many students in $mathscr{D}$ fail as possible.



To that end, my idea is to maximize the difference between the favorite students' grades and the pass/fail score, and also that between the pass/fail score and the disliked students' grades.



Note that we can use the total score as the pass/fail mark, instead of the average score. This means that in my solution, a student $i$ would pass if: $$sum_{j=1}^{12} frac{p_{i,j}w_j}{12} geq 0.6 sum_{j=1}^{12} 20w_j Rightarrow sum_{j=1}^{12} p_{i,j}w_j geq 12 sum_{j=1}^{12} w_j $$



(the 0.6 is from the condition that the pass/fail score is 12/20)



So we can set up the linear program as follows:



Let $|mathscr{L}| = m$ and $|mathscr{D}| = n$, where $m,n in mathbb{Z}^{+}$ and are given.



$$ text{max} left[left(sum_{i:S_i in mathscr{L}; 1 leq j leq 12} p_{i,j}w_jright) - left(12msum_{1 leq j leq 12}w_jright)right] + left[left( 12nsum_{1 leq j leq 12}w_jright) - left(sum_{i:S_i in mathscr{D}; 1 leq j leq 12} p_{i,j}w_jright)right]$$



$$text{subject to:} w_j geq 0, text{where} 1leq j leq 12$$










share|cite|improve this question











$endgroup$

















    1












    $begingroup$


    Please have a look at my solution to the following problem.



    enter image description here



    Firstly, I think that there are two ways to interpret this problem. If we need to make sure all students in $mathscr{L}$ pass and all students in $mathscr{D}$ fail, this problem doesn't need to be a linear program since the constraints are already enough to determine the solutions (i.e. the average grades of the favorite students are at least 60%, and those of the disliked students less than 60%).



    But there are of course situations where the above constraints could not all be met (e.g. a favorite student scoring lower than a disliked student in all 12 assignments), so I think the objective of the program should be to choose the weights so that as many students in $mathscr{L}$ pass as possible, and as many students in $mathscr{D}$ fail as possible.



    To that end, my idea is to maximize the difference between the favorite students' grades and the pass/fail score, and also that between the pass/fail score and the disliked students' grades.



    Note that we can use the total score as the pass/fail mark, instead of the average score. This means that in my solution, a student $i$ would pass if: $$sum_{j=1}^{12} frac{p_{i,j}w_j}{12} geq 0.6 sum_{j=1}^{12} 20w_j Rightarrow sum_{j=1}^{12} p_{i,j}w_j geq 12 sum_{j=1}^{12} w_j $$



    (the 0.6 is from the condition that the pass/fail score is 12/20)



    So we can set up the linear program as follows:



    Let $|mathscr{L}| = m$ and $|mathscr{D}| = n$, where $m,n in mathbb{Z}^{+}$ and are given.



    $$ text{max} left[left(sum_{i:S_i in mathscr{L}; 1 leq j leq 12} p_{i,j}w_jright) - left(12msum_{1 leq j leq 12}w_jright)right] + left[left( 12nsum_{1 leq j leq 12}w_jright) - left(sum_{i:S_i in mathscr{D}; 1 leq j leq 12} p_{i,j}w_jright)right]$$



    $$text{subject to:} w_j geq 0, text{where} 1leq j leq 12$$










    share|cite|improve this question











    $endgroup$















      1












      1








      1





      $begingroup$


      Please have a look at my solution to the following problem.



      enter image description here



      Firstly, I think that there are two ways to interpret this problem. If we need to make sure all students in $mathscr{L}$ pass and all students in $mathscr{D}$ fail, this problem doesn't need to be a linear program since the constraints are already enough to determine the solutions (i.e. the average grades of the favorite students are at least 60%, and those of the disliked students less than 60%).



      But there are of course situations where the above constraints could not all be met (e.g. a favorite student scoring lower than a disliked student in all 12 assignments), so I think the objective of the program should be to choose the weights so that as many students in $mathscr{L}$ pass as possible, and as many students in $mathscr{D}$ fail as possible.



      To that end, my idea is to maximize the difference between the favorite students' grades and the pass/fail score, and also that between the pass/fail score and the disliked students' grades.



      Note that we can use the total score as the pass/fail mark, instead of the average score. This means that in my solution, a student $i$ would pass if: $$sum_{j=1}^{12} frac{p_{i,j}w_j}{12} geq 0.6 sum_{j=1}^{12} 20w_j Rightarrow sum_{j=1}^{12} p_{i,j}w_j geq 12 sum_{j=1}^{12} w_j $$



      (the 0.6 is from the condition that the pass/fail score is 12/20)



      So we can set up the linear program as follows:



      Let $|mathscr{L}| = m$ and $|mathscr{D}| = n$, where $m,n in mathbb{Z}^{+}$ and are given.



      $$ text{max} left[left(sum_{i:S_i in mathscr{L}; 1 leq j leq 12} p_{i,j}w_jright) - left(12msum_{1 leq j leq 12}w_jright)right] + left[left( 12nsum_{1 leq j leq 12}w_jright) - left(sum_{i:S_i in mathscr{D}; 1 leq j leq 12} p_{i,j}w_jright)right]$$



      $$text{subject to:} w_j geq 0, text{where} 1leq j leq 12$$










      share|cite|improve this question











      $endgroup$




      Please have a look at my solution to the following problem.



      enter image description here



      Firstly, I think that there are two ways to interpret this problem. If we need to make sure all students in $mathscr{L}$ pass and all students in $mathscr{D}$ fail, this problem doesn't need to be a linear program since the constraints are already enough to determine the solutions (i.e. the average grades of the favorite students are at least 60%, and those of the disliked students less than 60%).



      But there are of course situations where the above constraints could not all be met (e.g. a favorite student scoring lower than a disliked student in all 12 assignments), so I think the objective of the program should be to choose the weights so that as many students in $mathscr{L}$ pass as possible, and as many students in $mathscr{D}$ fail as possible.



      To that end, my idea is to maximize the difference between the favorite students' grades and the pass/fail score, and also that between the pass/fail score and the disliked students' grades.



      Note that we can use the total score as the pass/fail mark, instead of the average score. This means that in my solution, a student $i$ would pass if: $$sum_{j=1}^{12} frac{p_{i,j}w_j}{12} geq 0.6 sum_{j=1}^{12} 20w_j Rightarrow sum_{j=1}^{12} p_{i,j}w_j geq 12 sum_{j=1}^{12} w_j $$



      (the 0.6 is from the condition that the pass/fail score is 12/20)



      So we can set up the linear program as follows:



      Let $|mathscr{L}| = m$ and $|mathscr{D}| = n$, where $m,n in mathbb{Z}^{+}$ and are given.



      $$ text{max} left[left(sum_{i:S_i in mathscr{L}; 1 leq j leq 12} p_{i,j}w_jright) - left(12msum_{1 leq j leq 12}w_jright)right] + left[left( 12nsum_{1 leq j leq 12}w_jright) - left(sum_{i:S_i in mathscr{D}; 1 leq j leq 12} p_{i,j}w_jright)right]$$



      $$text{subject to:} w_j geq 0, text{where} 1leq j leq 12$$







      proof-verification linear-programming






      share|cite|improve this question















      share|cite|improve this question













      share|cite|improve this question




      share|cite|improve this question








      edited Jan 13 at 20:27







      ensbana

















      asked Jan 13 at 14:52









      ensbanaensbana

      268113




      268113






















          1 Answer
          1






          active

          oldest

          votes


















          1












          $begingroup$

          I think you're absolutely right in your overall analysis that there are two different ways to interpret the question, and that, in the first way, the problem may be unsolvable. However, I'm pretty sure you should be analyzing it in the first way anyway.



          The conditions that each liked student passes, and each disliked student fails, are all themselves linear constraints which you can write down. So that gives you a linear program to solve (with trivial objective function, or any objective function you like). It is possible that in some cases the linear program will have no feasible solutions. This is fine! It just means that the answer in those cases is "there is no way for the professor to do this."



          The solution you've given, as an attempt to solve the second interpretation of the question, has two problems. One is that you haven't specified a scale for the $w_j$. So if you have any feasible solution where the value of the objective function is positive, you can get another one where it's twice as big just by doubling the value of every $w_j$. You could fix this by adding some additional constraint on the $w_j$ like forcing their sum to be $1$.



          The second problem is harder to deal with. Essentially, you're requiring the favored students to pass by as much as possible and the disfavored students to fail by as much as possible. But that doesn't guarantee that the number of students who have the right outcome is large. It might just be that you've chosen weights which are really good for some specific favored student (or really bad for some specific disfavored student) while being relatively neutral for everyone else. Another way to think about this is that the question is asking about outcomes in a pass/fail system, and your solution optimizes for situations where grades are a lot more granular (like one where percentages are reported).



          Fundamentally the issue is that the objective function you actually want under the second interpretation is "number of students with the correct outcome." This is nonlinear (and discontinuous!), so you can't use a linear program to maximize it. This is why I believe that the first interpretation is the correct one.



          If you wanted to solve the second interpretation of the problem, you could do it by removing students from $mathcal{L}$ and $mathcal{D}$ one by one until you got a feasible solution, but this would be computationally expensive — there are a lot of combinations to try. It is unlikely that there's a rapid way to solve the second interpretation; this kind of problem with integer variables is usually NP-hard.






          share|cite|improve this answer









          $endgroup$













          • $begingroup$
            Thank you for the very thorough comment! I actually thought about both issues that you mentioned before coming up with the solution above. In addressing the first problem, let (1),(2),(3),(4) be the four terms in parentheses in my objective function, plus the signs in front of them. If you increase any of the $w_j$'s, then (1) would increase, (4) would decrease, and (2) and (3) together could move either way depending on the cardinality of $m$ and $n$. My point is that there are terms in the expression that move in opposite ways, so shouldn't this ensure that $w_j$'s wouldn't go to infinity?
            $endgroup$
            – ensbana
            Jan 13 at 22:18










          • $begingroup$
            As for the second problem, (1) and (2) could be seen as the sum of $m$ terms, corresponding to $m$ favored students. You are right in saying that tweaking the $w_j$'s would mean some on those students are favored more than the others in the same group, but the thing is when you increase the weight of a particular assignment, the grades in that assignment of all disliked student also increase, thereby the terms in (3) combined with (4) would decrease.
            $endgroup$
            – ensbana
            Jan 13 at 22:26










          • $begingroup$
            So the program should choose weights so that the "spreads" around the pass/fail mark are as large as possible, and since those "spreads" are defined differently for the two groups of students, I hope that this would translate to the number of students passed or failed.
            $endgroup$
            – ensbana
            Jan 13 at 22:28










          • $begingroup$
            Okay I might have just seen where the problem is. If things move like I described, then yes this solution would make sense. But maybe the terms don't move like that, since their rates of change don't conform to the way I think I modeled them. There might be cases where, by keep increasing some of the $w_j$'s, the grades of the favored students would increase quite fast, the pass/fail grade also increases but not as fast, and the grades of the disliked students increases quite slowly. Then we might have both problems that you specified.
            $endgroup$
            – ensbana
            Jan 13 at 22:42












          • $begingroup$
            Also I’m quite interested in learning about the integer program that you mentioned. Could you elaborate a bit more on the approach, what the variables are and what the objective function looks like? Thanks.
            $endgroup$
            – ensbana
            Jan 14 at 1:12













          Your Answer





          StackExchange.ifUsing("editor", function () {
          return StackExchange.using("mathjaxEditing", function () {
          StackExchange.MarkdownEditor.creationCallbacks.add(function (editor, postfix) {
          StackExchange.mathjaxEditing.prepareWmdForMathJax(editor, postfix, [["$", "$"], ["\\(","\\)"]]);
          });
          });
          }, "mathjax-editing");

          StackExchange.ready(function() {
          var channelOptions = {
          tags: "".split(" "),
          id: "69"
          };
          initTagRenderer("".split(" "), "".split(" "), channelOptions);

          StackExchange.using("externalEditor", function() {
          // Have to fire editor after snippets, if snippets enabled
          if (StackExchange.settings.snippets.snippetsEnabled) {
          StackExchange.using("snippets", function() {
          createEditor();
          });
          }
          else {
          createEditor();
          }
          });

          function createEditor() {
          StackExchange.prepareEditor({
          heartbeatType: 'answer',
          autoActivateHeartbeat: false,
          convertImagesToLinks: true,
          noModals: true,
          showLowRepImageUploadWarning: true,
          reputationToPostImages: 10,
          bindNavPrevention: true,
          postfix: "",
          imageUploader: {
          brandingHtml: "Powered by u003ca class="icon-imgur-white" href="https://imgur.com/"u003eu003c/au003e",
          contentPolicyHtml: "User contributions licensed under u003ca href="https://creativecommons.org/licenses/by-sa/3.0/"u003ecc by-sa 3.0 with attribution requiredu003c/au003e u003ca href="https://stackoverflow.com/legal/content-policy"u003e(content policy)u003c/au003e",
          allowUrls: true
          },
          noCode: true, onDemand: true,
          discardSelector: ".discard-answer"
          ,immediatelyShowMarkdownHelp:true
          });


          }
          });














          draft saved

          draft discarded


















          StackExchange.ready(
          function () {
          StackExchange.openid.initPostLogin('.new-post-login', 'https%3a%2f%2fmath.stackexchange.com%2fquestions%2f3072065%2fchoosing-weights-of-assignments-to-ensure-that-some-particular-students-fail-wh%23new-answer', 'question_page');
          }
          );

          Post as a guest















          Required, but never shown

























          1 Answer
          1






          active

          oldest

          votes








          1 Answer
          1






          active

          oldest

          votes









          active

          oldest

          votes






          active

          oldest

          votes









          1












          $begingroup$

          I think you're absolutely right in your overall analysis that there are two different ways to interpret the question, and that, in the first way, the problem may be unsolvable. However, I'm pretty sure you should be analyzing it in the first way anyway.



          The conditions that each liked student passes, and each disliked student fails, are all themselves linear constraints which you can write down. So that gives you a linear program to solve (with trivial objective function, or any objective function you like). It is possible that in some cases the linear program will have no feasible solutions. This is fine! It just means that the answer in those cases is "there is no way for the professor to do this."



          The solution you've given, as an attempt to solve the second interpretation of the question, has two problems. One is that you haven't specified a scale for the $w_j$. So if you have any feasible solution where the value of the objective function is positive, you can get another one where it's twice as big just by doubling the value of every $w_j$. You could fix this by adding some additional constraint on the $w_j$ like forcing their sum to be $1$.



          The second problem is harder to deal with. Essentially, you're requiring the favored students to pass by as much as possible and the disfavored students to fail by as much as possible. But that doesn't guarantee that the number of students who have the right outcome is large. It might just be that you've chosen weights which are really good for some specific favored student (or really bad for some specific disfavored student) while being relatively neutral for everyone else. Another way to think about this is that the question is asking about outcomes in a pass/fail system, and your solution optimizes for situations where grades are a lot more granular (like one where percentages are reported).



          Fundamentally the issue is that the objective function you actually want under the second interpretation is "number of students with the correct outcome." This is nonlinear (and discontinuous!), so you can't use a linear program to maximize it. This is why I believe that the first interpretation is the correct one.



          If you wanted to solve the second interpretation of the problem, you could do it by removing students from $mathcal{L}$ and $mathcal{D}$ one by one until you got a feasible solution, but this would be computationally expensive — there are a lot of combinations to try. It is unlikely that there's a rapid way to solve the second interpretation; this kind of problem with integer variables is usually NP-hard.






          share|cite|improve this answer









          $endgroup$













          • $begingroup$
            Thank you for the very thorough comment! I actually thought about both issues that you mentioned before coming up with the solution above. In addressing the first problem, let (1),(2),(3),(4) be the four terms in parentheses in my objective function, plus the signs in front of them. If you increase any of the $w_j$'s, then (1) would increase, (4) would decrease, and (2) and (3) together could move either way depending on the cardinality of $m$ and $n$. My point is that there are terms in the expression that move in opposite ways, so shouldn't this ensure that $w_j$'s wouldn't go to infinity?
            $endgroup$
            – ensbana
            Jan 13 at 22:18










          • $begingroup$
            As for the second problem, (1) and (2) could be seen as the sum of $m$ terms, corresponding to $m$ favored students. You are right in saying that tweaking the $w_j$'s would mean some on those students are favored more than the others in the same group, but the thing is when you increase the weight of a particular assignment, the grades in that assignment of all disliked student also increase, thereby the terms in (3) combined with (4) would decrease.
            $endgroup$
            – ensbana
            Jan 13 at 22:26










          • $begingroup$
            So the program should choose weights so that the "spreads" around the pass/fail mark are as large as possible, and since those "spreads" are defined differently for the two groups of students, I hope that this would translate to the number of students passed or failed.
            $endgroup$
            – ensbana
            Jan 13 at 22:28










          • $begingroup$
            Okay I might have just seen where the problem is. If things move like I described, then yes this solution would make sense. But maybe the terms don't move like that, since their rates of change don't conform to the way I think I modeled them. There might be cases where, by keep increasing some of the $w_j$'s, the grades of the favored students would increase quite fast, the pass/fail grade also increases but not as fast, and the grades of the disliked students increases quite slowly. Then we might have both problems that you specified.
            $endgroup$
            – ensbana
            Jan 13 at 22:42












          • $begingroup$
            Also I’m quite interested in learning about the integer program that you mentioned. Could you elaborate a bit more on the approach, what the variables are and what the objective function looks like? Thanks.
            $endgroup$
            – ensbana
            Jan 14 at 1:12


















          1












          $begingroup$

          I think you're absolutely right in your overall analysis that there are two different ways to interpret the question, and that, in the first way, the problem may be unsolvable. However, I'm pretty sure you should be analyzing it in the first way anyway.



          The conditions that each liked student passes, and each disliked student fails, are all themselves linear constraints which you can write down. So that gives you a linear program to solve (with trivial objective function, or any objective function you like). It is possible that in some cases the linear program will have no feasible solutions. This is fine! It just means that the answer in those cases is "there is no way for the professor to do this."



          The solution you've given, as an attempt to solve the second interpretation of the question, has two problems. One is that you haven't specified a scale for the $w_j$. So if you have any feasible solution where the value of the objective function is positive, you can get another one where it's twice as big just by doubling the value of every $w_j$. You could fix this by adding some additional constraint on the $w_j$ like forcing their sum to be $1$.



          The second problem is harder to deal with. Essentially, you're requiring the favored students to pass by as much as possible and the disfavored students to fail by as much as possible. But that doesn't guarantee that the number of students who have the right outcome is large. It might just be that you've chosen weights which are really good for some specific favored student (or really bad for some specific disfavored student) while being relatively neutral for everyone else. Another way to think about this is that the question is asking about outcomes in a pass/fail system, and your solution optimizes for situations where grades are a lot more granular (like one where percentages are reported).



          Fundamentally the issue is that the objective function you actually want under the second interpretation is "number of students with the correct outcome." This is nonlinear (and discontinuous!), so you can't use a linear program to maximize it. This is why I believe that the first interpretation is the correct one.



          If you wanted to solve the second interpretation of the problem, you could do it by removing students from $mathcal{L}$ and $mathcal{D}$ one by one until you got a feasible solution, but this would be computationally expensive — there are a lot of combinations to try. It is unlikely that there's a rapid way to solve the second interpretation; this kind of problem with integer variables is usually NP-hard.






          share|cite|improve this answer









          $endgroup$













          • $begingroup$
            Thank you for the very thorough comment! I actually thought about both issues that you mentioned before coming up with the solution above. In addressing the first problem, let (1),(2),(3),(4) be the four terms in parentheses in my objective function, plus the signs in front of them. If you increase any of the $w_j$'s, then (1) would increase, (4) would decrease, and (2) and (3) together could move either way depending on the cardinality of $m$ and $n$. My point is that there are terms in the expression that move in opposite ways, so shouldn't this ensure that $w_j$'s wouldn't go to infinity?
            $endgroup$
            – ensbana
            Jan 13 at 22:18










          • $begingroup$
            As for the second problem, (1) and (2) could be seen as the sum of $m$ terms, corresponding to $m$ favored students. You are right in saying that tweaking the $w_j$'s would mean some on those students are favored more than the others in the same group, but the thing is when you increase the weight of a particular assignment, the grades in that assignment of all disliked student also increase, thereby the terms in (3) combined with (4) would decrease.
            $endgroup$
            – ensbana
            Jan 13 at 22:26










          • $begingroup$
            So the program should choose weights so that the "spreads" around the pass/fail mark are as large as possible, and since those "spreads" are defined differently for the two groups of students, I hope that this would translate to the number of students passed or failed.
            $endgroup$
            – ensbana
            Jan 13 at 22:28










          • $begingroup$
            Okay I might have just seen where the problem is. If things move like I described, then yes this solution would make sense. But maybe the terms don't move like that, since their rates of change don't conform to the way I think I modeled them. There might be cases where, by keep increasing some of the $w_j$'s, the grades of the favored students would increase quite fast, the pass/fail grade also increases but not as fast, and the grades of the disliked students increases quite slowly. Then we might have both problems that you specified.
            $endgroup$
            – ensbana
            Jan 13 at 22:42












          • $begingroup$
            Also I’m quite interested in learning about the integer program that you mentioned. Could you elaborate a bit more on the approach, what the variables are and what the objective function looks like? Thanks.
            $endgroup$
            – ensbana
            Jan 14 at 1:12
















          1












          1








          1





          $begingroup$

          I think you're absolutely right in your overall analysis that there are two different ways to interpret the question, and that, in the first way, the problem may be unsolvable. However, I'm pretty sure you should be analyzing it in the first way anyway.



          The conditions that each liked student passes, and each disliked student fails, are all themselves linear constraints which you can write down. So that gives you a linear program to solve (with trivial objective function, or any objective function you like). It is possible that in some cases the linear program will have no feasible solutions. This is fine! It just means that the answer in those cases is "there is no way for the professor to do this."



          The solution you've given, as an attempt to solve the second interpretation of the question, has two problems. One is that you haven't specified a scale for the $w_j$. So if you have any feasible solution where the value of the objective function is positive, you can get another one where it's twice as big just by doubling the value of every $w_j$. You could fix this by adding some additional constraint on the $w_j$ like forcing their sum to be $1$.



          The second problem is harder to deal with. Essentially, you're requiring the favored students to pass by as much as possible and the disfavored students to fail by as much as possible. But that doesn't guarantee that the number of students who have the right outcome is large. It might just be that you've chosen weights which are really good for some specific favored student (or really bad for some specific disfavored student) while being relatively neutral for everyone else. Another way to think about this is that the question is asking about outcomes in a pass/fail system, and your solution optimizes for situations where grades are a lot more granular (like one where percentages are reported).



          Fundamentally the issue is that the objective function you actually want under the second interpretation is "number of students with the correct outcome." This is nonlinear (and discontinuous!), so you can't use a linear program to maximize it. This is why I believe that the first interpretation is the correct one.



          If you wanted to solve the second interpretation of the problem, you could do it by removing students from $mathcal{L}$ and $mathcal{D}$ one by one until you got a feasible solution, but this would be computationally expensive — there are a lot of combinations to try. It is unlikely that there's a rapid way to solve the second interpretation; this kind of problem with integer variables is usually NP-hard.






          share|cite|improve this answer









          $endgroup$



          I think you're absolutely right in your overall analysis that there are two different ways to interpret the question, and that, in the first way, the problem may be unsolvable. However, I'm pretty sure you should be analyzing it in the first way anyway.



          The conditions that each liked student passes, and each disliked student fails, are all themselves linear constraints which you can write down. So that gives you a linear program to solve (with trivial objective function, or any objective function you like). It is possible that in some cases the linear program will have no feasible solutions. This is fine! It just means that the answer in those cases is "there is no way for the professor to do this."



          The solution you've given, as an attempt to solve the second interpretation of the question, has two problems. One is that you haven't specified a scale for the $w_j$. So if you have any feasible solution where the value of the objective function is positive, you can get another one where it's twice as big just by doubling the value of every $w_j$. You could fix this by adding some additional constraint on the $w_j$ like forcing their sum to be $1$.



          The second problem is harder to deal with. Essentially, you're requiring the favored students to pass by as much as possible and the disfavored students to fail by as much as possible. But that doesn't guarantee that the number of students who have the right outcome is large. It might just be that you've chosen weights which are really good for some specific favored student (or really bad for some specific disfavored student) while being relatively neutral for everyone else. Another way to think about this is that the question is asking about outcomes in a pass/fail system, and your solution optimizes for situations where grades are a lot more granular (like one where percentages are reported).



          Fundamentally the issue is that the objective function you actually want under the second interpretation is "number of students with the correct outcome." This is nonlinear (and discontinuous!), so you can't use a linear program to maximize it. This is why I believe that the first interpretation is the correct one.



          If you wanted to solve the second interpretation of the problem, you could do it by removing students from $mathcal{L}$ and $mathcal{D}$ one by one until you got a feasible solution, but this would be computationally expensive — there are a lot of combinations to try. It is unlikely that there's a rapid way to solve the second interpretation; this kind of problem with integer variables is usually NP-hard.







          share|cite|improve this answer












          share|cite|improve this answer



          share|cite|improve this answer










          answered Jan 13 at 21:57









          MicahMicah

          30k1364106




          30k1364106












          • $begingroup$
            Thank you for the very thorough comment! I actually thought about both issues that you mentioned before coming up with the solution above. In addressing the first problem, let (1),(2),(3),(4) be the four terms in parentheses in my objective function, plus the signs in front of them. If you increase any of the $w_j$'s, then (1) would increase, (4) would decrease, and (2) and (3) together could move either way depending on the cardinality of $m$ and $n$. My point is that there are terms in the expression that move in opposite ways, so shouldn't this ensure that $w_j$'s wouldn't go to infinity?
            $endgroup$
            – ensbana
            Jan 13 at 22:18










          • $begingroup$
            As for the second problem, (1) and (2) could be seen as the sum of $m$ terms, corresponding to $m$ favored students. You are right in saying that tweaking the $w_j$'s would mean some on those students are favored more than the others in the same group, but the thing is when you increase the weight of a particular assignment, the grades in that assignment of all disliked student also increase, thereby the terms in (3) combined with (4) would decrease.
            $endgroup$
            – ensbana
            Jan 13 at 22:26










          • $begingroup$
            So the program should choose weights so that the "spreads" around the pass/fail mark are as large as possible, and since those "spreads" are defined differently for the two groups of students, I hope that this would translate to the number of students passed or failed.
            $endgroup$
            – ensbana
            Jan 13 at 22:28










          • $begingroup$
            Okay I might have just seen where the problem is. If things move like I described, then yes this solution would make sense. But maybe the terms don't move like that, since their rates of change don't conform to the way I think I modeled them. There might be cases where, by keep increasing some of the $w_j$'s, the grades of the favored students would increase quite fast, the pass/fail grade also increases but not as fast, and the grades of the disliked students increases quite slowly. Then we might have both problems that you specified.
            $endgroup$
            – ensbana
            Jan 13 at 22:42












          • $begingroup$
            Also I’m quite interested in learning about the integer program that you mentioned. Could you elaborate a bit more on the approach, what the variables are and what the objective function looks like? Thanks.
            $endgroup$
            – ensbana
            Jan 14 at 1:12




















          • $begingroup$
            Thank you for the very thorough comment! I actually thought about both issues that you mentioned before coming up with the solution above. In addressing the first problem, let (1),(2),(3),(4) be the four terms in parentheses in my objective function, plus the signs in front of them. If you increase any of the $w_j$'s, then (1) would increase, (4) would decrease, and (2) and (3) together could move either way depending on the cardinality of $m$ and $n$. My point is that there are terms in the expression that move in opposite ways, so shouldn't this ensure that $w_j$'s wouldn't go to infinity?
            $endgroup$
            – ensbana
            Jan 13 at 22:18










          • $begingroup$
            As for the second problem, (1) and (2) could be seen as the sum of $m$ terms, corresponding to $m$ favored students. You are right in saying that tweaking the $w_j$'s would mean some on those students are favored more than the others in the same group, but the thing is when you increase the weight of a particular assignment, the grades in that assignment of all disliked student also increase, thereby the terms in (3) combined with (4) would decrease.
            $endgroup$
            – ensbana
            Jan 13 at 22:26










          • $begingroup$
            So the program should choose weights so that the "spreads" around the pass/fail mark are as large as possible, and since those "spreads" are defined differently for the two groups of students, I hope that this would translate to the number of students passed or failed.
            $endgroup$
            – ensbana
            Jan 13 at 22:28










          • $begingroup$
            Okay I might have just seen where the problem is. If things move like I described, then yes this solution would make sense. But maybe the terms don't move like that, since their rates of change don't conform to the way I think I modeled them. There might be cases where, by keep increasing some of the $w_j$'s, the grades of the favored students would increase quite fast, the pass/fail grade also increases but not as fast, and the grades of the disliked students increases quite slowly. Then we might have both problems that you specified.
            $endgroup$
            – ensbana
            Jan 13 at 22:42












          • $begingroup$
            Also I’m quite interested in learning about the integer program that you mentioned. Could you elaborate a bit more on the approach, what the variables are and what the objective function looks like? Thanks.
            $endgroup$
            – ensbana
            Jan 14 at 1:12


















          $begingroup$
          Thank you for the very thorough comment! I actually thought about both issues that you mentioned before coming up with the solution above. In addressing the first problem, let (1),(2),(3),(4) be the four terms in parentheses in my objective function, plus the signs in front of them. If you increase any of the $w_j$'s, then (1) would increase, (4) would decrease, and (2) and (3) together could move either way depending on the cardinality of $m$ and $n$. My point is that there are terms in the expression that move in opposite ways, so shouldn't this ensure that $w_j$'s wouldn't go to infinity?
          $endgroup$
          – ensbana
          Jan 13 at 22:18




          $begingroup$
          Thank you for the very thorough comment! I actually thought about both issues that you mentioned before coming up with the solution above. In addressing the first problem, let (1),(2),(3),(4) be the four terms in parentheses in my objective function, plus the signs in front of them. If you increase any of the $w_j$'s, then (1) would increase, (4) would decrease, and (2) and (3) together could move either way depending on the cardinality of $m$ and $n$. My point is that there are terms in the expression that move in opposite ways, so shouldn't this ensure that $w_j$'s wouldn't go to infinity?
          $endgroup$
          – ensbana
          Jan 13 at 22:18












          $begingroup$
          As for the second problem, (1) and (2) could be seen as the sum of $m$ terms, corresponding to $m$ favored students. You are right in saying that tweaking the $w_j$'s would mean some on those students are favored more than the others in the same group, but the thing is when you increase the weight of a particular assignment, the grades in that assignment of all disliked student also increase, thereby the terms in (3) combined with (4) would decrease.
          $endgroup$
          – ensbana
          Jan 13 at 22:26




          $begingroup$
          As for the second problem, (1) and (2) could be seen as the sum of $m$ terms, corresponding to $m$ favored students. You are right in saying that tweaking the $w_j$'s would mean some on those students are favored more than the others in the same group, but the thing is when you increase the weight of a particular assignment, the grades in that assignment of all disliked student also increase, thereby the terms in (3) combined with (4) would decrease.
          $endgroup$
          – ensbana
          Jan 13 at 22:26












          $begingroup$
          So the program should choose weights so that the "spreads" around the pass/fail mark are as large as possible, and since those "spreads" are defined differently for the two groups of students, I hope that this would translate to the number of students passed or failed.
          $endgroup$
          – ensbana
          Jan 13 at 22:28




          $begingroup$
          So the program should choose weights so that the "spreads" around the pass/fail mark are as large as possible, and since those "spreads" are defined differently for the two groups of students, I hope that this would translate to the number of students passed or failed.
          $endgroup$
          – ensbana
          Jan 13 at 22:28












          $begingroup$
          Okay I might have just seen where the problem is. If things move like I described, then yes this solution would make sense. But maybe the terms don't move like that, since their rates of change don't conform to the way I think I modeled them. There might be cases where, by keep increasing some of the $w_j$'s, the grades of the favored students would increase quite fast, the pass/fail grade also increases but not as fast, and the grades of the disliked students increases quite slowly. Then we might have both problems that you specified.
          $endgroup$
          – ensbana
          Jan 13 at 22:42






          $begingroup$
          Okay I might have just seen where the problem is. If things move like I described, then yes this solution would make sense. But maybe the terms don't move like that, since their rates of change don't conform to the way I think I modeled them. There might be cases where, by keep increasing some of the $w_j$'s, the grades of the favored students would increase quite fast, the pass/fail grade also increases but not as fast, and the grades of the disliked students increases quite slowly. Then we might have both problems that you specified.
          $endgroup$
          – ensbana
          Jan 13 at 22:42














          $begingroup$
          Also I’m quite interested in learning about the integer program that you mentioned. Could you elaborate a bit more on the approach, what the variables are and what the objective function looks like? Thanks.
          $endgroup$
          – ensbana
          Jan 14 at 1:12






          $begingroup$
          Also I’m quite interested in learning about the integer program that you mentioned. Could you elaborate a bit more on the approach, what the variables are and what the objective function looks like? Thanks.
          $endgroup$
          – ensbana
          Jan 14 at 1:12




















          draft saved

          draft discarded




















































          Thanks for contributing an answer to Mathematics Stack Exchange!


          • Please be sure to answer the question. Provide details and share your research!

          But avoid



          • Asking for help, clarification, or responding to other answers.

          • Making statements based on opinion; back them up with references or personal experience.


          Use MathJax to format equations. MathJax reference.


          To learn more, see our tips on writing great answers.




          draft saved


          draft discarded














          StackExchange.ready(
          function () {
          StackExchange.openid.initPostLogin('.new-post-login', 'https%3a%2f%2fmath.stackexchange.com%2fquestions%2f3072065%2fchoosing-weights-of-assignments-to-ensure-that-some-particular-students-fail-wh%23new-answer', 'question_page');
          }
          );

          Post as a guest















          Required, but never shown





















































          Required, but never shown














          Required, but never shown












          Required, but never shown







          Required, but never shown

































          Required, but never shown














          Required, but never shown












          Required, but never shown







          Required, but never shown







          Popular posts from this blog

          Mario Kart Wii

          What does “Dominus providebit” mean?

          Antonio Litta Visconti Arese